MathGroup Archive 1997

[Date Index] [Thread Index] [Author Index]

Search the Archive

Re: wrong divergence?!?

  • To: mathgroup at smc.vnet.net
  • Subject: [mg6407] Re: wrong divergence?!?
  • From: wexler at u.washington.edu (Carlos Wexler)
  • Date: Sun, 16 Mar 1997 19:25:24 -0500 (EST)
  • Organization: University of Washington, Seattle
  • Sender: owner-wri-mathgroup at wolfram.com

In article <5gdg3l$kmd at smc.vnet.net>,
Edward G. Neuman <edneuman at math.siu.edu> wrote:
>
>
>peter <psalzman at landau.ucdavis.edu> wrote in article
><5gagic$g02 at smc.vnet.net>...
>> 
>> hi all
>> 
>> when i take the divergence of:
>> 
>>   e = {q/r^2, 0, 0}
>> 
>> i get zero.  i remembered to load VectorAnalysis and SetCoordinates to 
>> Spherical.
>> 
>> this is the electric field due to a point charge at the origin.  even 
>> though MMA doesn't know maxwell's equations, if we strip all the physical
>
>> meaning from this vector field, shouldn't the divergence be something
>other 
>> than zero at the origin?
>> 
>> thanks!
>> 
>> peter
>> --
>> Birthdays are good for you:  A federal funded project has recently
>determined
>> that people with the most number of birthdays will live the longest.....
>>
>-=><=-+-+-=><=-+-+-=><=-+-+-=><=-+-+-=><=-+-+-=><=-+-+-=><=-+-+-=><=-+-+-=><
>=-
>>      I BOYCOTT ANY COMPANY THAT USES MASS ADVERTISING ON THE INTERNET
>> 
>> 
>> 
>
>Function Div returns a correct value. To check it calculate divergence in
>spherical coordinates to obtain:
>
>div (q/r^2, 0 , 0) = (1/r^2) * D[(r^2) * (q/r^2), r] = (1/r^2) * D[q,r] =
>0.
>I assume that q does not depend on r.
>
>I hope this helps.
>
>
>
>====================================================
>Edward Neuman
>E-mail: edneuman at siu.edu
>http://www.math.siu.edu/neuman/personal.html

No, the divergence of (1/r^2, 0, 0) is the Dirac delta function (times
some constantes that I don't remember)...

Carlos


-- 






  • Prev by Date: Re: Re: Problems with TimeConstrained in Win3.1
  • Next by Date: system of equations
  • Previous by thread: Re: Re: wrong divergence?!?
  • Next by thread: Integration error with Chebyshev polynomials